Difference between revisions of "1967 AHSME Problems/Problem 26"

(Created page with "== Problem == If one uses only the tabular information <math>10^3=1000</math>, <math>10^4=10,000</math>, <math>2^{10}=1024</math>, <math>2^{11}=2048</math>, <math>2^{12}=4096</ma...")
 
m (See also)
 
(5 intermediate revisions by 2 users not shown)
Line 9: Line 9:
  
 
== Solution ==
 
== Solution ==
<math>\fbox{C}</math>
+
Since <math>1024</math> is greater than <math>1000</math>.
 +
 
 +
<math>\log (1024) > 3</math>   
 +
 
 +
<math>10 * \log (2) > 3</math>
 +
 
 +
and <math>\log (2) > 3/10</math>.
 +
 
 +
 
 +
Similarly, <math>8192 < 10000</math>, so <math>\log (8192) < 4</math>
 +
 
 +
<math>13 * \log (2) < 4</math>
 +
 
 +
and <math>\log (2) < 4/13</math>
 +
 
 +
 
 +
Therefore  <math>3/10 < \log 2 < 4/13</math>
 +
so the answer is <math>\fbox{C}</math>
  
 
== See also ==
 
== See also ==
{{AHSME box|year=1967|num-b=25|num-a=27}}   
+
{{AHSME 40p box|year=1967|num-b=25|num-a=27}}   
  
 
[[Category:Introductory Algebra Problems]]
 
[[Category:Introductory Algebra Problems]]
 
{{MAA Notice}}
 
{{MAA Notice}}

Latest revision as of 01:40, 16 August 2023

Problem

If one uses only the tabular information $10^3=1000$, $10^4=10,000$, $2^{10}=1024$, $2^{11}=2048$, $2^{12}=4096$, $2^{13}=8192$, then the strongest statement one can make for $\log_{10}{2}$ is that it lies between:

$\textbf{(A)}\ \frac{3}{10} \; \text{and} \; \frac{4}{11}\qquad \textbf{(B)}\ \frac{3}{10} \; \text{and} \; \frac{4}{12}\qquad \textbf{(C)}\ \frac{3}{10} \; \text{and} \; \frac{4}{13}\qquad \textbf{(D)}\ \frac{3}{10} \; \text{and} \; \frac{40}{132}\qquad \textbf{(E)}\ \frac{3}{11} \; \text{and} \; \frac{40}{132}$

Solution

Since $1024$ is greater than $1000$.

$\log (1024) > 3$

$10 * \log (2) > 3$

and $\log (2) > 3/10$.


Similarly, $8192 < 10000$, so $\log (8192) < 4$

$13 * \log (2) < 4$

and $\log (2) < 4/13$


Therefore $3/10 < \log 2 < 4/13$ so the answer is $\fbox{C}$

See also

1967 AHSC (ProblemsAnswer KeyResources)
Preceded by
Problem 25
Followed by
Problem 27
1 2 3 4 5 6 7 8 9 10 11 12 13 14 15 16 17 18 19 20 21 22 23 24 25 26 27 28 29 30 31 32 33 34 35 36 37 38 39 40
All AHSME Problems and Solutions

The problems on this page are copyrighted by the Mathematical Association of America's American Mathematics Competitions. AMC logo.png